I Why are there 2s -1 independent integrals of motion?

AI Thread Summary
The discussion revolves around the concept of independent integrals of motion in mechanics, specifically addressing confusion regarding the number of initial conditions required. It highlights that while 2N initial conditions are typically specified, the distinction between position and velocity is questioned, suggesting that they should be treated separately. The Lagrangian's independence from acceleration is emphasized, leading to a discussion about the implications for equations of motion. The need for time independence in autonomous systems is also examined, particularly regarding the relationship between time and position variables. Overall, the conversation seeks clarity on the mathematical foundations of motion integrals and their implications in mechanics.
yucheng
Messages
232
Reaction score
57
I was reading Mechanics by Landau and Lifshitz and I am confused when it is stated in chapter 2 section 6 that one of the integrals of motion is not independent and it can be considered an additive constant of time. Hence I tried searching it up online.

https://physics.stackexchange.com/q...f-motion-vs-first-integrals?noredirect=1&lq=1

According to the OP in the link above (first paragraph second sentence), we need to specify 2N initial conditions, one of them is the initial time, the others the initial positions and velocity.

However, shouldn't it be N position and N velocities? Can it be shown to be equivalent? Plus aren't we working with an autonomous system of equations? Is this why we need ##t - t_0## so that it is independent of time?

https://physics.stackexchange.com/questions/13832/integrals-of-motion

The answer provided above seems interesting. However, how correct is it? There are several points that I would like to verify...

Questions:
  1. Because the ##\mathcal{L} (q, \dot q)##, the Lagrangian is independent of the acceleration. Hence $$\frac{d}{dt} \frac{\partial \mathcal{L} }{\partial \dot q} - \frac{\partial \mathcal{L}}{\partial q}$$
    which only involved one time derivative, only introduces terms linear in ##\ddot q##.
  2. According to the author (see a comment below the post as well), $$\ddot q_i =\frac{\text{d}\dot q_i}{\text{d} q_1} \frac{\text{d}q_1}{\text{d}t} =\dot q_1\frac{\text{d}\dot q_i}{\text{d}q_1}$$. How does a total time derivative become a total derivative in ##q_1##? Are we performing a change of variables by inverting ##q_1(t)## to get time as a function of ##q_1## then all coordinates become ##q_i(t(q_1))##?
  3. Are there other proofs of this?
 
Last edited:
Physics news on Phys.org
yucheng said:
However, shouldn't it be N position and N velocities? Can it be shown to be equivalent? Plus aren't we working with an autonomous system of equations? Is this why we need t−t0 so that it is independent of time?
Hi. Position and velocity change in time in general so they cannot be integral constants e.g. energy.
 
anuttarasammyak said:
Hi. Position and velocity change in time in general so they cannot be integral constants e.g. energy.
Actually, for that part of the question, I am referring to the initial conditions (which of course determines the ##2s## integrals of motions). I asked it here because it is very relevant! ;)
 
Ooops by the way, ##N=s## (degrees of freedom)
 
yucheng said:
Because the L(q,q˙), the Lagrangian is independent of the acceleration. Hence $$\frac{d}{dt} \frac{\partial \mathcal{L} }{\partial \dot q} - \frac{\partial \mathcal{L}}{\partial q}$$
which only involved one time derivative, only introduces terms linear in q¨.
Is this correct?
 
The rope is tied into the person (the load of 200 pounds) and the rope goes up from the person to a fixed pulley and back down to his hands. He hauls the rope to suspend himself in the air. What is the mechanical advantage of the system? The person will indeed only have to lift half of his body weight (roughly 100 pounds) because he now lessened the load by that same amount. This APPEARS to be a 2:1 because he can hold himself with half the force, but my question is: is that mechanical...
Let there be a person in a not yet optimally designed sled at h meters in height. Let this sled free fall but user can steer by tilting their body weight in the sled or by optimal sled shape design point it in some horizontal direction where it is wanted to go - in any horizontal direction but once picked fixed. How to calculate horizontal distance d achievable as function of height h. Thus what is f(h) = d. Put another way, imagine a helicopter rises to a height h, but then shuts off all...
Some physics textbook writer told me that Newton's first law applies only on bodies that feel no interactions at all. He said that if a body is on rest or moves in constant velocity, there is no external force acting on it. But I have heard another form of the law that says the net force acting on a body must be zero. This means there is interactions involved after all. So which one is correct?
Back
Top